Intégrale(s) de printemps 2022

Fin de partie
Modifié (March 2022) dans Analyse
Je vous propose de montrer que $\displaystyle \int_0^1 \dfrac{x-1}{(1+x)^3\ln x}dx=\dfrac{7\zeta(3)}{24\zeta(2)}$

Réponses

  • bd2017
    Modifié (March 2022)
    Bonjour
    Changement de variable $ x=\exp(-u)$  puis  intégrale de $-\infty$    à  $+\infty$ (la fonction est paire) ensuite  théorème des résidus  avec un contour de la forme d'un demi-disque mais en contournant 0.
    Il me semble que ça marche mais je regarde le foot et je n'ai pas le temps de vérifier.
     
  • On peut calculer l'intégrale proposée sans variable complexe.
    Une indication:
    Considérer l'intégrale $\displaystyle \int_0^\infty \dfrac{x-1}{(1+x)^3\ln x}dx$

    La suite du calcul est très classique d'une certaine manière.
  • jmf
    jmf
    Modifié (April 2022)
    $x=1/t$
  • Fin de partie
    Modifié (April 2022)
    @JMF: Si tu fais ce changement de variable dans la deuxième intégrale cela ne change pas l'intégrande, ni les bornes d'intégration.
    Pour moi, le rapport entre les deux intégrales proposées est évident mais encore fallait-il y penser, car autrement on ne peut pas continuer avec les bornes d'intégration $0$ et $1$. Le changement de bornes, mais sans changer l'intégrande, ouvre des perspectives utiles pour la suite du calcul. On peut montrer que: \begin{align}\displaystyle \int_0^\infty \dfrac{x-1}{(1+x)^3\ln x}dx=2\int_0^1 \dfrac{x-1}{(1+x)^3\ln x}dx\end{align} mais le reste du calcul est un peu moins évident.

    PS:
    Pour continuer on a besoin d'un ingrédient classique et d'utiliser un truc qui a déjà été utilisé sur ce forum: Il faut se débarrasser du logarithme au dénominateur qui est une gêne pour la suite du calcul.
  • Écrire $\frac{x-1}{\ln(x)}=\int_0^1e^{t\ln(x)}dt$ doit pas mal simplifier les choses.
    Ensuite, j'essaierais de faire un développement en série entière de $\frac{1}{1+x^3}$ et le changement de variable $x=e^{-u}$ et deux interversions ... mais ça n'a pas l'air de parfaitement converger tout cela.
  • jmf
    jmf
    Modifié (April 2022)

    @Fin de partie

    Je faisais le changement de variable dans la première intégrale pour me ramener à la seconde....

    Dans $I=\displaystyle\int_{0}^{1}\!\dfrac{x-1}{(1+x)^3\ln x}\,\text{d}x$, on pose $x=1/t$.

    On obtient $I=\displaystyle\int_{1}^{+\infty}\!\!\!\dfrac{1/t-1}{t^2(1+1/t)^3(-\ln t)}\,\text{d}t=\displaystyle\int_{1}^{+\infty}\!\!\!\dfrac{t-1}{(1+t)^3\ln t}\,\text{d}t$, donc $I=\dfrac{1}{2}\displaystyle\int_{0}^{+\infty}\!\!\!\dfrac{x-1}{(1+x)^3\ln x}\,\text{d}x$.

    On pose maintenant $x=\text{e}^{2t}$. On trouve : $I=\dfrac{1}{2}\displaystyle\int_{-\infty}^{+\infty}\!\!\!\dfrac{\text{e}^{2t}-1}{t(1+\text{e}^{2t})^3}\text{e}^{2t}\,\text{d}t$.

    Mais $\dfrac{\text{e}^{2t}-1}{(1+\text{e}^{2t})^3}\text{e}^{2t}=\dfrac{\text{e}^{3t}\bigl(\text{e}^t-\text{e}^{-t}\bigr)}{\text{e}^{3t}\big(\text{e}^t+\text{e}^{-t}\bigr)^3}=\dfrac{1}{4}\dfrac{\text{sh}\,t}{(\text{ch}\,t)^3}$

    À ce stade, on a donc $I=\dfrac{1}{8}\displaystyle\int_{-\infty}^{+\infty}\!\!\!\dfrac{\text{sh}\,t}{t\,(\text{ch}\,t)^3}\,\text{d}t=\dfrac{1}{8}\displaystyle\int_{-\infty}^{+\infty}\dfrac{1}{t}\text{th}'t\;\text{th}\,t\,\text{d}t$.

    On intègre par parties : $I=\dfrac{1}{16}\displaystyle\int_{-\infty}^{+\infty}\Bigl(\dfrac{\text{th}\,t}{t}\Bigr)^2\,\text{d}t=\dfrac{1}{8}\displaystyle\int_{0}^{+\infty}\Bigl(\dfrac{\text{th}\,t}{t}\Bigr)^2\,\text{d}t$.

    Et maintenant, que vais-je faireuh ?

  • @Bisam: tu as suivi, en partie, la même piste que moi. Tu as mis sur la table l'un des ingrédients manquants. Pas besoin de développement en série.

    @Jmf:  mis à part le changement de bornes de l'intégrale, je n'ai pas emprunté le même chemin que toi je ne saurais dire si c'est une piste viable ou une impasse.
  • @Fin de partie

    Avant que ce fil ne sombre dans un oubli immérité, pourrait-on avoir une petite indication?
  • Fin de partie
    Modifié (April 2022)
    @JMF je vais faire la synthèse de ce qui permet d'aller vers une solution.

    1) \begin{align} \int_0^1 \dfrac{x-1}{(1+x)^3\ln x}dx=\frac{1}{2}\int_0^\infty \dfrac{x-1}{(1+x)^3\ln x}dx\end{align}
    2) Pour $x>0$, \begin{align} \frac{x-1}{\ln x}=-\int_0^1 x^s ds\end{align}

    Commence déjà par tirer les conséquences de tout ceci et, avec un tout petit plus de connaissances, la suite coule de source.

    PS:
    Je suis arrivé assez rapidement à l'expression qui fait la synthèse de tout ceci mais j'avais les "mauvaises" bornes d'intégration, $0$ et $1$ ce qui fait que je ne voyais pas comment continuer.

    PS2:
    La suite coule de source mais on se ramène à calculer une intégrale qui n'est tout de même pas évidente. Mais l'intégrale en question est traitée sur MathExchange. J'en avais donné un calcul qui est compliqué. Je me suis rendu compte à l'occasion qu'on pouvait faire bien plus simple et c'était à ma portée à l'époque mais j'ai raté ce calcul.
  • On intègre par parties : $\dfrac{1}{8}\displaystyle\int_{0}^{+\infty}\Bigl(\dfrac{\text{th}\,t}{t}\Bigr)^2\,\text{d}t$.

    Et maintenant, que vais-je faireuh ?

    Cette dernière intégrale est fait là 

    https://math.stackexchange.com/questions/1582943/integral-int-0-infty-frac-tanh2xx2dx?noredirect=1


  • jmf
    jmf
    Modifié (April 2022)
    Ok merci @etanche

    Parmi un florilège d'approches différentes, j'extrais cette partie (sans doute la plus accessible).

    $$I=\int_{-\infty}^{\infty} \frac{\tanh ^{2} x}{2 x^{2}} d x \stackrel{t=1 / e^{2 x}}{=} \int_{0}^{\infty}\left(\frac{1-t}{1+t}\right)^{2} \frac{d t}{t \ln ^{2} t} \stackrel{i b p}{=} 4 \int_{0}^{\infty} \frac{t-1}{(1+t)^{3}} \frac{d t}{\ln t}$$
    Let $J(a)=4\displaystyle \int_{0}^{\infty} \frac{t^{a}-1}{(1+t)^{3}} \frac{d t}{\ln t}$. Then, $J^{\prime}(a)=4\displaystyle \int_{0}^{\infty} \frac{t^{a}}{(1+t)^{3}} d t=\frac{2 \pi a(1-a)}{\sin \pi a}$ and
    $$I=\int_{0}^{1} J^{\prime}(a) d a \stackrel{i b p}{=} \int_{0}^{1}(4 a-2) \ln \tan \frac{\pi a}{2} d a \stackrel{t=\tan \frac{\pi a}{2}}{=} \frac{4}{\pi^{2}} \int_{0}^{\infty} \frac{\ln t \tan ^{-1} \sqrt{t}}{\sqrt{t}(1+t)} d t$$
    Let $K(a)=\displaystyle\int_{0}^{\infty} \frac{\ln t \tan ^{-1} a \sqrt{t}}{\sqrt{t}(1+t)} d t$. Then, $K^{\prime}(a)=\displaystyle\int_{0}^{\infty} \frac{\ln t}{(1+t)\left(1+a^{2} t\right)} d t=\frac{2 \ln ^{2} a}{1-a^{2}}$ and $$I=\frac{4}{\pi^{2}} \int_{0}^{1} K^{\prime}(a) d a=\frac{8}{\pi^{2}} \int_{0}^{1} \frac{\ln ^{2} a}{1-a^{2}} d a=\frac{8}{\pi^{2}} \cdot \frac{7}{4} \zeta(3)=\frac{14 \zeta(3)}{\pi^{2}}$$
    PS: comment on fait pour remonter jusqu'à cette discussion sur stackexchange?
  • Fin de partie
    Modifié (April 2022)
    C'est à peu près mon approche du calcul mais le calcul recopié omet un point important qui n'est pas détaillé. 
    Le point clef est l'égalité (qui est fausse):
    \begin{align}\int_{0}^{\infty} \frac{t^{a}}{(1+t)^{3}} d t=\frac{2 \pi a(1-a)}{\sin (\pi a)}\end{align}
    qui demande quelques connaissances.
    PS.

    La bonne formule est
    \begin{align}\int_{0}^{\infty} \frac{t^{a}}{(1+t)^{3}} d t=\frac{ \pi a(1-a)}{2\sin (\pi a)}\end{align} qui est valable pour $0\leq a\leq 1$
    Merci à Jean Lismonde pour sa vigilance.

  • Fin de partie
    Modifié (April 2022)
    Le même gars qui est l'auteur du calcul recopié ci-dessus propose une idée* qui est intéressante mais sa rédaction est peu esthétique pour moi.
    On a besoin de calculer l'intégrale $\displaystyle \int_0^\infty \dfrac{\ln x\arctan(x)}{1+x^2}dx$
    * c'est un changement de variable mais qui peut être remplacé par une intégration par parties dans le cas d'espèce mais j'espère qu'on peut l'utiliser dans d'autres intégrales où il serait incontournable pour un calcul rapide.
  • Fin de partie
    Modifié (April 2022)
    \begin{align}K&=\int_0^\infty \frac{\arctan u\ln u}{1+u^2}du\\ &=\int_0^1 \left(\int_0^\infty \frac{u\ln u}{(1+u^2)(1+t^2u^2)}du\right)dt\\&=\overset{z(u)=\frac{1}{tu}}=-\int_0^1 \left(\int_0^\infty \frac{z\ln (tz)}{(1+z^2)(1+t^2z^2)}dz\right)dt\\&=-K-\int_0^1 \left(\int_0^\infty \frac{z}{(1+z^2)(1+t^2z^2)}dz\right)\ln t dt\\&=-K+\frac{1}{2}\int_0^1 \left [\frac{\ln\left(\frac{1+t^2z^2}{1+z^2}\right)}{1-t^2} \right ]_0^\infty\ln t dt\\&=-K+\int_0^1 \frac{\ln^2 t}{1-t^2}dt\\&=-K+\int_0^1 \frac{\ln^2 t}{1-t}dt-\underbrace{\int_0^1 \frac{t\ln^2 t}{1-t^2}dt}_{u=t^2}\\&=-K+\frac{7}{8}\int_0^1 \frac{\ln^2 t}{1-t}dt\\ &=\frac{7}{16}\int_0^1 \frac{\ln^2 t}{1-t}dt\\ &=\frac{7}{16}\times 2\zeta(3)\\ &=\boxed{\frac{7}{8}\zeta(3)} \end{align}
    PS. Je voulais explorer ce que donne un tel changement de changement de variable ; sorti de derrière les fagots, sur des intégrales du type : \begin{align}\int_0^\infty \frac{u^\delta\ln u}{(1+u^\beta)(1+t^\alpha u^\gamma)}du\end{align}
    Le but est que le changement de variable ne modifie que l'intérieur du logarithme.
  • Bonjour FDP

    ta relation $\int_0^{+\infty}\frac{t^a}{(1+t)^3}dt = \frac{2\pi.a}{sin(\pi.a)}(1-a)$       est fausse pour a =0

    en effet le premier membre donne 1/2 alors que le second membre donne 2

    dans l'expression originelle de l'intégrale, le 2 se situerait plutôt au dénominateur, qu'en penses-tu ?

    Cordialement
  • Fin de partie
    Modifié (April 2022)
    @Jean Lismonde: Je pense qu'on s'en fiche ici car $a=0$ est une borne de l'intégrale double.

    PS: Je regarde s'il y a un problème dans cette formule.

    PS2: @Jean Lismonde: Numériquement tu sembles avoir raison.
  • Fin de partie
    Modifié (April 2022)
    Pour $0 \leq a \leq 1$,
    \begin{align}\int_0^\infty \frac{t^a}{(1+t)^3}dt&=\text{B}\left(1+a,2-a\right)\\
    &\frac{\Gamma\left(1+a\right)\Gamma\left(2-a\right)}{\Gamma\left(1+a+2-a\right)}\\
    &=\frac{a\Gamma(a)\times (1-a)\Gamma(1-a)}{\Gamma(3)}\\
    &=\frac{a(1-a)\overbrace{\Gamma(a)\Gamma(1-a)}^{=\frac{\pi}{\sin\left(\pi a\right)}}}{2}\\
    &=\boxed{\frac{\pi a(1-a)}{2\sin\left(\pi a\right)}}
    \end{align}
    NB. $\text{B}$ est la fonction Bêta d'Euler.
Connectez-vous ou Inscrivez-vous pour répondre.